Prepare an accounting equation for the following business transaction

1. Purchased supplies on account, 4,000

2. Purchased furniture and fixtures for cash, 3,000

3. Paid account in no. 1.

4. Paid advance advertisement for promoting the business 1,000

5. Paid the note 2,000

Answers

Answer 1

Answer:

you draw a T. form and then you can start to in debit and credit


Related Questions

geomtry plz help 15 points

Answers

Answer:

it should be 60°. the single marks on each side means they are all equal

Answer:

m∠L=60º

Step-by-step explanation:

The triangle is an equilateral triangle meaning that all angles and sides are equal.

m∠L=60º

Hope this helps :)

35. In her rough work, Janu added 2389 and 740 as shown here.
Is it okay to write the number 740 this way in the addition?
(Hint: You do NOT need to calculate)
(A) Yes, because 0 has no value so 74 is the same as 740.
(B) No, because the answer will be wrong if 0 is not written.
(C) No, because the digits 7 and 4 have NOT been arranged according to their value.
(D) Yes, because 7 is added with the hundred's digit and 4 with the ten's digit.

Answers

Answer:

D

Step-by-step explanation:

plz help on a timed test will pay 10 points and make brainlest

What other countries have set up early earthquake warning systems? Which of these countries are in the Ring of Fire? Why is it important for these Ring of Fire countries to have warning systems?

Answers

Answer:

Mexico, Japan, South Korea, Taiwan, and the United States, Chile, Japan, the US west coast;  Chile, Japan, the US west coast; So all that is left for the people who live around the Ring of Fire is to be aware of the danger, perhaps to live further inland, build safer, earthquake-resistant housing, and for nations everywhere to improve oceanic and land-based early-warning systems to help minimize the risk to life

Step-by-step explanation:

Suppose a jar contains 17 red marbles and 32 blue marbles. If you reach in the jar and pull out 2 marbles at random, find the probability that both are red.

Answers

There are 49 total marbles, the chance of you getting a red is 17/49, and if you don’t replace the marble and pull out another one, the probability of that being red is 17/49*16/48=17/147. That’s approximately a .12% chance.
16/48=1/3

The probability that both are red marbles = [tex]\bold{\frac{17}{147} }[/tex]

What is probability?

"It is finding out the possibilities of the occurrence of an event."

Formula to find the probability of an event:

"P(A) = n(A) / n(S)

where, n(A) is the number of favorable outcomes of an event A

n(S) is the total number of outcomes for an experiment"

For given example,

A jar contains 17 red marbles and 32 blue marbles.

If we pull out 2 marbles at random, we need to find the probability that both are red.

The number of possible outcomes,

[tex]\Rightarrow n(S)=^{49}C_2\\\\ \Rightarrow n(S)=\frac{49!}{2!(49-2)!}\\\\ \Rightarrow n(S)=1176[/tex]

The number of possible outcomes of selecting both the red marbles.

[tex]\Rightarrow n(A)=^{17}C_2\\\\ \Rightarrow n(A)=\frac{17!}{2!(17-2)!}\\\\ \Rightarrow n(A)=136[/tex]

The probability that both the marbles are red,

[tex]\Rightarrow P(A)=\frac{n(A)}{n(S)}\\\\ \Rightarrow P(A)=\frac{136}{1176}\\\\ \Rightarrow P(A)=\frac{17}{147}[/tex]

Therefore, the probability that both are red marbles = [tex]\bold{\frac{17}{147} }[/tex]

Learn more about the probability here:

brainly.com/question/11234923

#SPJ2

geomtry plzzz help 15 points

Answers

Answer:

19

Step-by-step explanation:

they give you xz, and they want half, so just divide 38 by 2

geomtry plz help 15 points

Answers

Answer:

m∠O = 41°

Step-by-step explanation:

∠NOM=∠NMO=(4y-15)° (base angles of isos triangle)

7y+2(4y-15)=180 (angle sum of triangle)

  7y+8y-30=180

       15y-30=180

             15y=180+30

                  =210

                y=210÷15

                  =14

Hence, m∠O = (4y-15)°

                      = [4(14)-15]°

                      = (56-15)°

                      = 41°

Juan wants to buy a video game for $63. He saves $12 every Friday. Part A. Create an equation to represent Juan's total savings, y, in dollars, after x Fridays. y​

Answers

Answer:

Answer in explanation.

Step-by-step explanation:

An equation to represent Juan's total savings y, in dollars, after x Fridays will be;

⇒ y = 12x

What is an expression?

Mathematical expression is defined as the collection of the numbers variables and functions by using operations like addition, subtraction, multiplication, and division.

Given that;

Juan wants to buy a video game for $63.

And, He saves $12 every Friday.

Now,

Let total dollars after x Friday = y

And, He saves $12 every Friday.

So, We can formulate;

⇒ Total money (y) = 12x

⇒ y = 12x

Thus, An equation to represent Juan's total savings y, in dollars, after x Fridays will be;

⇒ y = 12x

Learn more about the mathematical expression visit:

brainly.com/question/1859113

#SPJ2

What is 75.65 rounded to the nearest whole number?​

Answers

It is 76 as the other person said, what I like to think to help me remember is five or more raise the score, 4 or less, let it rest

31 divided by 943.89

Answers

Answer:

0.032

Step-by-step explanation:

Answer:  0.03284281007

Step-by-step explanation:

if a chocolate cake is divided into six equal slices and each slice has 280 calories

Answers

Answer:

Whats the question?

Step-by-step explanation:

Answer:

1,680

Step-by-step explanation

6 slices each 280 calories, 280 x 6 = 1,680

2x - 5y = 6 and -2x + 7y = 14

*Using elimination*​

Answers

2x-5y+ (-2x) + 7y= 6+14
2y = 20
y= 10
2x-5(10)=6
2x= 56
x= 28
here is the ans.hope this helps!

g A long-term study has revealed that a test for cancer in men is very effective. The study shows that 89% of the men for which the test is positive actually have cancer. If a man selected at random tests positive for cancer with this test, what is the probability that he does not have cancer

Answers

The answer is 89% because I had that Same question and it was 89%

Solve the equation: 3.017 + k = 5.134

Answers

Answer:

k=2.117

Step-by-step explanation:

Answer:

3.017 + k = 5.134

So, k = 5.134 - 3.017

So, k = 2.117

The width of a triangle is six more than twice the height. The area of the triangle is 88in2. Find the height and width of the triangle.

Answers

Answer:

w = 22 in, h = 8 in.

Step-by-step explanation:

Reading the problem, I immediately see two equations. This hints me towards a system of equations problem.

Statement 1 is The width of a triangle is six more than twice the height.

The width (w) of a triangle is (=) six (6) more (+) than twice (2) the height (h).

Equation 1: w = 6 + 2h

Statement 2 is The area of the triangle is 88 in2.

The formula for the area of a triangle is [tex]\frac{wh}{2}[/tex]. So, we get our second equation.

Equation 2: wh/2 = 88

We see that we have the same number of equations as unknowns, two. This is important as it lets us know that we can find two unknowns in the two equations. If there are more unknowns than equations, then we cannot find the values of all the variables. Look for another equation in the problem. However, if the # equations greater than or equal to number of unknowns, then we are ready to start solving!  So, we can now move to solving.

Equation 1 isolates one of the unknowns (w). So, we can replace w in the second equation with the right side of eq 1. Then, we simplify for h.

[tex]\frac{wh}{2} = 88[/tex]

[tex]\frac{(6 + 2h)h}{2} = 88[/tex]

[tex]\frac{6h + 2h^2}{2} = 88[/tex]

[tex]6h + 2h^2 = 176[/tex]

[tex]2h^2 + 6h - 176 = 0[/tex]

[tex]h = -11, 8[/tex] (assuming you know how to solve quadratics)

Since we know that heights of triangles cannot be negative, we find that [tex]h = 8[/tex] in.

For the final step, we plug [tex]h[/tex] into our first equation to find [tex]w[/tex].

[tex]w = 6 + 2h[/tex]

[tex]w = 6 + 2(8)[/tex]

[tex]w = 6 + 16[/tex]

[tex]w = 22[/tex]

So, the answer is w = 22 in, h = 8 in.

To check if we were right, we can plug the values into both equations to see if they are equal. I will skip equation 1 since we used that equation "as-is" to find w.

[tex]\frac{wh}{2} = 88[/tex]

[tex]\frac{22 * 8}{2} = 88[/tex]

[tex]11 * 8 = 88[/tex]

[tex]88 = 88[/tex]

The solution checks out!

please help this is for my study guide thanks! ​

Answers

The volume is 29.32 <3

which number will reach if we move 6 steps to the right -3

Answers

Answer:

If you meant -3 + 6, then the correct answer is 3.

You should put some extra effort in composing questions with all the required information, like a picture maybe.

Answer:

your answer has to be 3

Step-by-step explanation:

if you were to move to the right in number line you have to add so -3+6 is equals to 3

Last year at a certain high school, there were 100 boys on the honor roll and 60 girls on the honor roll. This year, the number of boys on the honor roll decreased by 11% and the number of girls on the honor roll decreased by 10%. By what percentage did the total number of students on the honor roll increase? Round your answer to the nearest tenth (if necessary).​

Answers

Answer:

-11 boys -6 girls

Step-by-step explanation:

Evaluate this function: h(t) = |t+2| + 3; Find h(6)

Answers

Answer:

Step-by-step explanation:

h(6) = |6 + 2| + 3 = |8| + 3 = 8 + 3 = 11

Jason gets 15 gallons of gas for $3.90 a gallon which gives him 20 miles to the gallon. How many miles can Jason travel on one tank of Jason?

Answers

0.75 miles
15 divided by 20 as its in proportion

Answer:

0.75 mi.

Step-by-step explanation:

15/20=3.90/x

how to find the volume of an oven with the inside dimensions 3 ft by 1.5 ft by 3 ft

Answers

9514 1404 393

Answer:

  13.5 ft³

Step-by-step explanation:

The volume of a cuboid is the product of its length, width, and height:

  V = (3 ft)(1.5 ft)(3 ft) = 13.5 ft³

The volume of the oven is 13.5 ft³.

__

You may see this as the formula ...

  V = LWH

what is 4% of 32? is it a percent, base, or amount

Answers

Answer:

1.28 - amount

Step-by-step explanation:

You would multiply 32 times 0.04 and get 1.28 which I assume would be an amount.

The amount of 4% of 32 is 1.28.

We have to determine what is 4% of 32.

What is the percentage?

This free percentage calculator computes a number of values involving percentages, including the percentage difference between two given values.

Suppose it is an x percent.

Therefore x multiply 32 times 0.04 and get 1.28 an amount.

To learn more about the percentage of visits:

https://brainly.com/question/24304697

#SPJ2

Which graph represents 6x-2y>-11

Answers

Answer:

Answer:

C

Step-by-step explanation:

Remember when using the inequality sign < or > the line will be dotted so we can immediately eliminate answer choices b and d

Now let us solve for y

6x-2y>-11

step 1 subtract each side by 6x

now we have

-2y>-11-6x

step 2 divide each side by -2

-11-/2=11/2

-6/-2=3

IMPORTANT -when you divide inequalities by a negative number you flip the inequality sign

So now we have

See image below for reference

When the inequality is facing x the solutions are going to be on top of the line

hope this helps!

Step-by-step explanation:


A high school track team's long jump record is 22 ft 6 3/4in. This year, Arthur's best long jump is 21 ft 10 1/2in. If long jumps are measured to the nearest quarter inch, how much farther must
Arthur jump to break the record?
To beat the school long jump record,

Arthur must jump an additional ___inches.

(Type a whole number, fraction, or mixed number.)

Answers

Arthur must jump an additional 8.25 inches so as to break the record.

1 foot =  12 inches

Long jump record = 22 ft 6 3/4in = 22 ft 6.75 in = (22ft * 12 in per ft) + 6.75 in

Long jump record = 270.75 in

Arthur's long jump record = 21 ft 10 1/2in = 21 ft 10.5 in = (21ft * 12 in per ft) + 10.5 in

Arthur's long jump record = 262.5 ft.

To break the record, the distance needed to be jump by Arthur = 270.75 in - 262.5 ft. = 8.25 in

Arthur must jump an additional 8.25 inches so as to break the record.

Find out more at: https://brainly.com/question/20796404

in a supermarket, credit card purchases have an additional charge of 8% of the purchase made. If I bought 2 suitcases that cost s / .165.60 each and paid with a credit card, how much do I have to pay?

Answers

9514 1404 393

Answer:

  357.70

Step-by-step explanation:

The final cost will be ...

  (number of suitcases) × (cost per suitcase) × (1 +8%)

  = 2(165.60)(1.08) = 357.70

Write the equation of the function graphed below:

a. f(x) = 2x−−√3 –4
b. f(x) = 2x−−√3 + 4
c. f(x) =x−−√3 + 4
d. f(x) =x−−√3 – 4

Answers

Answer:

I have a couple questions before I answer, what do the 2 dashes represent and are the 4s also under the square root?

Note: Enter your answer and show all the steps that you use to solve this problem in the space provided.

You buy clothing at a sale. You buy a sweater at
7
10
of its original price of $35. Answer each question and show all your work.

a. How much money did you spend?
b. How much money did you save?
c. What fraction of the total original price did you save?

Answers

The amount that will be spent is $24.50, the amount saved will be $10.50, and the percentage saved will be 30%.

The amount that will be spent will be:

= 7/10 × $35 = $24.50

The amount that will be saved will be:

= $35 - $24.50 = $10.50

The fraction of the original price that is saved will be:

= 10.50/35 × 100

= 30%

Read related link on:

https://brainly.com/question/25763817

How many pounds are in 1 1/2 pounds and 8 ounces? There are _____ pounds in 1 1/2 pounds and 8 ounces.

Answers

Answer:

Hope this helps

Step-by-step explanation:

Since 16 ounces equal 1 pound, we’ll add 16 + 16, which is 32 ounces. For one half of a pound, simply divide the amount of ounces in a pound (16) by half. 16 divided by 2 is 8, so in total your answer would be 40 ounces in 2 1/2 pounds.

What is the coefficient of x in expression of 5mx​

Answers

Coefficient of this expression is 5 only.

- BRAINLIEST answerer

e/kaid_274404147475337871026587/assig!
Khan Academy
Percent word problems
Of the 50 U.S. states, 4 have names that start with the letter W.
What percentage of U.S. states have names that start with the letter W?
0
Stuck? Review related articles/videos or use a hint.

Answers

Answer:

Step-by-step explanation:

4/50 = 0.08

0.08(100) = 8%

What’s the slope of 10,6 and 4, 1.2

Answers

let's firstly change the 1.2 to a fraction

[tex]1.\underline{2}\implies \cfrac{12}{1\underline{0}}\implies \cfrac{6}{5} \\\\[-0.35em] ~\dotfill\\\\ (\stackrel{x_1}{10}~,~\stackrel{y_1}{6})\qquad (\stackrel{x_2}{4}~,~\stackrel{y_2}{\frac{6}{5}}) \\\\\\ \stackrel{slope}{m}\implies \cfrac{\stackrel{rise} {\stackrel{y_2}{\frac{6}{5}}-\stackrel{y1}{6}}}{\underset{run} {\underset{x_2}{4}-\underset{x_1}{10}}}\implies \cfrac{~~ \frac{6-30}{5}~~}{-6}\implies \cfrac{~~ \frac{-24}{5}~~}{-6}\implies \cfrac{~~ -\frac{24}{5}~~}{-\frac{6}{1}}[/tex]

[tex]-\cfrac{\stackrel{4}{~~\begin{matrix} 24 \\[-0.7em]\cline{1-1}\\[-5pt]\end{matrix}~~}}{5}\cdot -\cfrac{1}{\underset{1}{~~\begin{matrix} 6 \\[-0.7em]\cline{1-1}\\[-5pt]\end{matrix}~~}}\implies \boxed{\cfrac{4}{5}}[/tex]

Other Questions
Summarize chapter 7 of the outsiders. Guys please help I suck at summarizing A tuning fork of frequency 254 Hz and an open orang pipe of slightly lower frequency are at 15oC. When sounded together, they produce 4 beats per second. On altering the temperature of the air in the pipes, it is observed that the number of beats per second first diminishes to zero and then increases again to 4. By how much has the temperature of the air in the pipe been altered? Please help!!Choose the definition for the function. What is the approximate solution to this? Comment on the effectiveness of the differing depictions of the main characters Ichabod Crane and King Prospero in the text as opposed to the movie. Respond to each depiction individually. what is politics called what event led to the end of operation reinhard? Help pls 10 pts.Drastic boundary changes can occur for all of the following reasons except for __________.A.a shift in the way a river flowsB.the collapse of a government systemC.a conflict between two neighboring groupsD.one countrys decision to sell land to another countryPlease select the best answer from the choices providedABCD What natural resources does mars have? Complete the Punnett square What was a result of the Whiskey Rebellion? A. The federal government asserted its authority. B. Farmers overthrew the state government in Pennsylvania. C.Washington gave up his role of commander in chief. D.The government agreed to immediately remove the tax on whiskey. my Englishhomework:'( Simplify (3x-9) / (x^2-x-6) i have to right about a jupiter sweet pepper and i have no clue I need help with these two questions!! HELP Zulu kingdom anti-colonial sentiment A car dealership pays a wholesale price of $12,000 to purchase a vehicle.The car dealership wants to make a 32% profit.After the markup, what is the retail price of the vehicle? Item 3Which statement is true of chloroplasts?They transport glucose into the cell.They are present in animal cells, but not in plant cells.They convert light energy into chemical energy.They convert the energy in glucose into a more usable form. what happens to the total mass of the atoms involved in a chemical reaction Following the liberation of Korea after World War II, which of the following took place?A: Korea became its own independent nation.B: The United States and USSR started a world war in Korea. C: A civil war broke out between north and south Korea.